Sei sulla pagina 1di 32

MASSIMI E MINIMI VINCOLATI

Esercizi risolti

1. Determinare il massimo e il minimo assoluti della funzione f (x, y) = xy sulla cir-


conferenza unitaria x2 + y 2 = 1.

2. Determinare il massimo e il minimo assoluti di f (x, y) = x2 + 3y con il vincolo


2
x2
4
+ y9 = 1.

3. Trovare il massimo e il minimo assoluti di f (x, y) = xy con il vincolo |x| + |y| = 1.

4. Trovare i punti a minima e massima distanza dall’origine della circonferenza di


equazione (x − 2)2 + (y − 1)2 = 1.

5. Trovare il massimo e il minimo assoluti di f (x, y) = x2 y lungo la curva x4 + y 4 = 1.

6. Trovare il massimo e il minimo assoluti di f (x, y) = |x| + |y| sulla curva x2 + y 2 = 1.


p
7. Studiare gli estremi vincolati della funzione f (x, y) = 2x2 − xy + y 2 lungo la
retta x + y = 8.

8. Sia y = f (x) l’equazione di una curva regolare e sia P = (a, b) un punto non ap-
partenente ad essa. Detto Q = (x, f (x)) un punto variabile sulla curva, dimostrare
che la retta P Q è perpendicolare alla curva se e solo se la distanza P Q è minima o
massima.

9. Determinare il punto Q sulla retta y = mx + q alla minima distanza dal punto


P = (a, b). Dimostrare poi che la distanza di P dalla retta, cioè la distanza P Q, è

|ma + q − b|
d = PQ = √ .
1 + m2

10. Trovare i punti a minima e massima distanza dall’origine della curva di equazione
x2 − xy + y 2 = 1.

11. Determinare il minimo di f (x, y) = x2 + y 2 con il vincolo (x − 1)3 − y 2 = 0.

12. Studiare gli estremi vincolati della funzione f (x, y) = −x log x−y log y con il vincolo
x + y = 1.

13. Studiare gli estremi vincolati della funzione f (x, y) = x + y con il vincolo xy = 1,
x > 0, y > 0.
x2 y2
14. Tra tutti i rettangoli inscritti nell’ellisse di equazione a2
+ b2
= 1 trovare quello di
perimetro massimo.

15. Tra tutte le ellissi di semiassi x, y tali che x2 +y 2 = 5 trovare quella di area massima.

16. Tra tutti i cilindri inscritti nella sfera di raggio R trovare quello di volume massimo.

1
17. Tra tutti i cilindri (circolari retti) di superficie totale S fissata (incluse le due facce
circolari) trovare quello di volume massimo.
18. Trovare il massimo e il minimo assoluti della funzione f (x, y, z) = xy + xz + yz
sulla sfera x2 + y 2 + z 2 = 1.
19. Trovare il massimo e il minimo assoluti della funzione f (x, y, z) = (x + y + z)2 con
il vincolo x2 + 2y 2 + 3z 2 = 1.
20. Tra tutti i parallelepipedi (rettangoli) di volume assegnato V determinare quello di
superficie minima.
21. Trovare i punti a minima e massima distanza dall’origine dell’ellisse ottenuto
2 2
tagliando l’ellissoide x4 + y4 + z 2 = 1 con il piano x + y + z = 1.
22. Tra tutti i triangoli di perimetro assegnato 2p trovare quello di area massima.
23. a) Trovare il minimo della funzione f (x1 , x2 , . . . , xn ) = x1 + x2 + · · · + xn con il
vincolo x1 · x2 · · · xn = k (k fissato > 0) e xj > 0 ∀j.
b) Dedurre la seguente disuguaglianza tra media geometrica e aritmetica: dati n
numeri positivi x1 , x2 , . . . , xn vale
√ x1 + x2 + · · · + xn
n
x1 x2 · · · xn ≤
n
con uguaglianza se e solo se x1 = x2 = · · · = xn .
24. Calcolare il massimo e il minimo assoluti della funzione f (x, y) = |x| + |y|
sull’insieme A = {(x, y) ∈ R2 : x2 + y 2 ≤ 1}. (Si ricordi l’es. 6.)
2 −y 2
25. Trovare il massimo e il minimo assoluti della funzione f (x, y) = ex sugli insiemi
a) A = {(x, y) ∈ R2 : x2 + y 2 ≤ 1};
b) B = {(x, y) ∈ R2 : |x| ≤ 1, |y| ≤ 1}.
26. Trovare il massimo e il minimo assoluti di f (x, y) = x2 y + xy 2 − xy nel triangolo
T = {(x, y) ∈ R2 : x + y ≤ 1, x ≥ 0, y ≥ 0}.
27. Trovare il massimo e il minimo assoluti di f (x, y) = x2 + y 2 − 3y + xy nel cerchio
A = {(x, y) ∈ R2 : x2 + y 2 ≤ 9}.

Esercizi di approfondimento

28. Consideriamo il problema di ottimizzare la funzione f (x, y, z) = xy + xz + yz


2 2 2
sull’ellissoide g(x, y, z) = xa2 + yb2 + zc2 − 1 = 0. (Si veda l’es. 18 dei risolti per il
caso a = b = c e l’es. 12 dei proposti per il caso a = b 6= c.) Dimostrare che il
minimo m = min f |g=0 e il massimo M = max f |g=0 sono soluzioni dell’equazione
di terzo grado

p(λ) = 4λ3 − a2 b2 + a2 c2 + b2 c2 λ − a2 b2 c2 = 0


2
e dedurre che m = min{λ : p(λ) = 0}, M = max{λ : p(λ) = 0}, procedendo come
segue:
a) scrivere il sistema ∇(f − λg)(x, y, z) = 0 come un sistema lineare omogeneo
3 × 3 nelle variabili (x, y, z), e dimostrare che il determinante della matrice A(λ)
del sistema vale − a2 b22 c2 p(λ). Dedurre che il sistema ha soluzioni non identicamente
nulle (x, y, z) 6= (0, 0, 0) se e solo se p(λ) = 0.
b) Moltiplicando la prima equazione per x, la seconda per y, la terza per z e
sommandole, dimostrare che se (x, y, z, λ) risolve il sistema
(
∇(f − λg)(x, y, z) = 0
(1)
g(x, y, z) = 0

allora λ = f (x, y, z), cioè il moltiplicatore di Lagrange coincide con il valore di f


nel punto (x, y, z).
c) Dimostrare che il polinomio p(λ) ha sempre 3 radici reali e distinte λ1 < λ2 <
0 < λ3 escluso il caso a = b = c in cui λ1 = λ2 < 0 < λ3 (vedi anche es. seguente).
A tal fine si studi la funzione p(λ), si verifichi che ha un massimo per un certo
valore λ− < 0, e si dimostri che p(λ− ) ≥ 0 utilizzando la disuguaglianza stabilita
nell’esercizio 23 con n = 3 e x1 = a2 b2 , x2 = a2 c2 , x3 = b2 c2 .
d) Dai punti a), b) e c) dedurre che m = λ1 , M = λ3 .

29. Nell’esercizio precedente sia a = b = c. Verificare che


2
a) λ1 = λ2 = − a2 , λ3 = a2 ;
b) per λ = λ3 la soluzione del sistema (1) è v3 = ( √a3 , √a3 , √a3 ) (unica a meno del
segno), mentre per λ = λ1 ogni vettore v = (x, y, z) di norma a e tale che x+y +z =
0 è soluzione, per esempio ogni combinazione lineare dei vettori v1 = ( √a2 , − √a2 , 0),
v2 = ( √a6 , √a6 , − √2a6 ).
Si confronti il risultato con l’es. 18 dei risolti.
2

a2 ±a a2 +8c2
30. Nell’esercizio 28 sia a = b 6= c. Verificare che le radici di p(λ) sono − a2 , 4
,
e dedurre che
√ ( 2
a2 + a a2 + 8c2 − a2 √ se a > c
M= , m= a2 −a a2 +8c2
4 4
se a < c.

Confrontare il risultato con l’es. 12 dei proposti.

31. Vogliamo usare i moltiplicatori di Lagrange per dimostrare il teorema sulla di-
agonalizzazione delle matrici simmetriche: sia A una matrice simmetrica reale
n × n; allora esiste una base ortonormale di Rn costituita di autovettori di A.
Consideriamo la forma
P quadratica associata ad A, cioè la funzione f : Rn → R,
f (x) = hx, Axi = Aij xi xj , e consideriamo il problema di ottimizzare f sulla sfera
unitaria S n−1 = {x ∈ Rn : kxk2 = 1}, cioè con il vincolo g(x) = kxk2 − 1 = 0.

3
La funzione f è continua e la restrizione f |S n−1 ammette massimo e minimo per il
teorema di Weierstrass (S n−1 è un sottoinsieme chiuso e limitato di Rn ).
a) Sia λ1 = max f |S n−1 e sia v1 ∈ S n−1 un punto di massimo, λ1 = f (v1 ). Di-
mostrare che Av1 = λ1 v1 , cioè v1 è autovettore di A con autovalore λ1 . (Si di-
mostrino le formule ∇g(x) = 2x, ∇f (x) = 2Ax.)
b) Si ottimizzi ora la funzione f sull’insieme

S n−2 = {x ∈ S n−1 : hx, v1 i = 0},

cioè con i due vincoli g(x) = 0 e h(x) = hx, v1 i = 0. Di nuovo f |S n−2 deve avere
massimo e minimo. Posto λ2 = max f |S n−2 e detto v2 ∈ S n−2 un punto di massimo,
λ2 = f (v2 ), si dimostri che Av2 = λ2 v2 facendo vedere che se (x, λ, µ) soddisfa il
sistema (
∇(f − λg − µh)(x) = 0
g(x) = 0, h(x) = 0
allora µ = 0, Ax = λx e λ = f (x), cioè il primo moltiplicatore di Lagrange è auto-
valore di A con autovettore x e coincide con f (x), mentre il secondo moltiplicatore
è zero.
c) Procedendo in modo analogo con l’insieme

S n−3 = {x ∈ S n−1 : hx, x1 i = 0, hx, x2 i = 0},

si ottenga Av3 = λ3 v3 , dove λ3 = max f |S n−3 = f (v3 ).


d) Iterando il procedimento concludere che si ottengono n vettori v1 , v2 , . . . , vn
ortogonali tra loro e di norma 1, cioè una base ortonormale di Rn , tali che Avk =
λk vk , dove λk = max f |S n−k = f (vk ), con λ1 ≥ λ2 ≥ · · · ≥ λn . Dedurre infine che
il massimo di f sulla sfera S n−1 è il più grande autovalore di A, il minimo di f
su S n−1 è il più piccolo autovalore di A, e i punti di massimo e di minimo sono i
corrispondenti autovettori (normalizzati).

32. Rivedere l’esercizio


 18 dei risolti
 scrivendo f come la forma quadratica associata
1 1
0 2 2
alla matrice A =  12 0 12  e applicando i risultati dell’esercizio precedente.
1 1
2 2
0

33. (Generalizzazione dell’esercizio


P 28). Sia A una matrice simmetrica reale n × n e
sia f (x) = hx, Axi = Aij xi xj la forma quadratica associata. Consideriamo il
problema di ottimizzare f sull’ellissoide
x21 x22 x2n
E n−1 = {x ∈ Rn : a21
+ a22
+ ··· + a2n
= 1},

P x2j
cioè con il vincolo g(x) = a2j
−1 = 0, dove a1 , a2 , . . . , an sono n numeri positivi as-
segnati (i semiassi dell’ellissoide). Sia B la matrice diagonale B = diag(a1 , a2 , . . . , an ).

4
a) Dimostrare che se (x, λ) risolve il sistema
(
∇(f − λg)(x) = 0
g(x) = 0

allora B 2 Ax = λx, cioè λ è autovalore di B 2 A con autovettore x, e inoltre λ = f (x).


b) Dimostrare che la matrice B 2 A (in generale non simmetrica) ha gli stessi auto-
valori della matrice simmetrica à = BAB e che y è autovettore di à con autovalore
λ se e solo se x = By è autovettore di B 2 A con autovalore λ.
c) PostoPx = By cioè xj = aj yj , verificare che g(x) = 0 se e solo se g̃(y) = 0, dove
g̃(y) = yj2 − 1, e che f (x) = hx, Axi = hy, Ãyi. Dedurre che
( (
∇(f − λg)(x) = 0 ∇(f˜ − λg̃)(y) = 0
se e solo se
g(x) = 0 g̃(y) = 0,

cioè ottimizzare f sull’ellissoide E n−1 equivale a ottimizzare la forma quadratica


f˜(y) = hy, Ãyi associata alla matrice à sulla sfera unitaria S n−1 . Applicando i
risultati dell’esercizio 31, concludere che

M = max f |E n−1 = max f˜|S n−1 e m = min f |E n−1 = min f˜|S n−1

coincidono rispettivamente con il massimo e il minimo autovalore di Ã.

34. Risolvere l’esercizio 12 dei proposti applicando i risultati dell’esercizio precedente.

35. Risolvere l’esercizio 19 dei risolti applicando i risultati dell’esercizio 33.

36. Riottenere i risultati dell’esercizio 28 dei risolti applicando i risultati dell’esercizio


33.

P3 A una matrice simmetrica reale 3 × 3 definita positiva. Sia g(x) = hx, Axi =
37. Sia
i,j=1 Aij xi xj la forma quadratica associata ad A e consideriamo l’ellissoide cen-
trato nell’origine di equazione g(x) = 1. Dimostrare che il semiasse minore (risp.
√ √
maggiore) dell’ellissoide è 1/ µ3 (risp. 1/ µ1 ), dove 0 < µ1 ≤ µ2 ≤ µ3 sono gli
autovalori di A. (Si ottimizzi la funzione f (x) = kxk2 con il vincolo g(x) = 1.)

5
SOLUZIONI

1. Determinare il massimo e il minimo assoluti della funzione f (x, y) = xy sulla cir-


conferenza unitaria x2 + y 2 = 1.
1o metodo. Applichiamo il metodo dei moltiplicatori di Lagrange, impostando il
sistema (
∇(f − λg)(x, y) = 0
(2)
g(x, y) = 0
dove il vincolo è dato da g(x, y) = x2 + y 2 − 1 = 0. Otteniamo

y = 2λx

x = 2λy

 2
x + y 2 = 1.

Sostituendo la prima equazione nella seconda si ha x = 4λ2 x, cioè x(1 − 4λ2 ) = 0.


Non può essere x = 0 perchè altrimenti anche y = 0 mentre (x, y) 6= (0, 0) per il
vincolo. Ne deduciamo che λ = ± 21 , da cui y = ±x. Sostituendo nel vincolo si
ottiene 2x2 = 1, da cui x = ± √12 . Otteniamo cosı̀ i 4 punti critici vincolati

P1 = ( √12 , √12 ), P2 = ( √12 , − √12 ), P3 = (− √12 , √12 ), P4 = (− √12 , − √12 ).

Poichè f è continua su S 1 = {(x, y) ∈ R2 : x2 + y 2 = 1}, che è un sottoinsieme


chiuso e limitato di R2 , f |S 1 deve ammettere massimo e minimo e i punti di estremo
vincolato vanno ricercati tra i punti Pj (1 ≤ j ≤ 4). Essendo f (P1 ) = f (P4 ) = 21 ,
f (P3 ) = f (P2 ) = − 12 , otteniamo
m = min f |S 1 = − 21 , M = max f |S 1 = 21 .

2o metodo. Possiamo parametrizzare il vincolo ponendo


(
x = cos t
y = sin t

e studiare la funzione di una variabile


h(t) = f (cos t, sin t) = cos t sin t = 12 sin 2t
sull’intervallo [0, 2π]. Disegnando il grafico di h è immediato verificare che h è
massima per t = π4 , 54 π, minima per t = 34 π, 74 π, e si riottiene m = − 12 , M = 21 .
Infine ricordiamo che se (x0 , y0 ) è un punto di estremo (massimo o minimo) per
f (x, y) vincolato a g(x, y) = 0 con ∇f (x0 , y0 ) 6= 0, ∇g(x0 , y0 ) 6= 0, allora la curva
di livello che passa per (x0 , y0 ), cioè f (x, y) = k0 con k0 = f (x0 , y0 ), è tangente alla
curva del vincolo g(x, y) = 0 nel punto (x0 , y0 ). Nel nostro caso le curve di livello
f (x, y) = xy = k sono iperboli equilatere con assi uguali agli coordinati, mentre
il vincolo è la circonferenza unitaria. Le iperboli f (x, y) = ± 21 sono tangenti alla
circonferenza nei punti P1 , P2 , P3 , P4 .

6
2. Determinare il massimo e il minimo assoluti di f (x, y) = x2 + 3y con il vincolo
x2 /4 + y 2 /9 = 1.
1o metodo. Con i moltiplicatori di Lagrange, scriviamo il sistema (2) con g(x, y) =
x2 /4 + y 2 /9 − 1: 
λ
2x = 2 x

3 = 92 λy
 x2 y 2

4
+ 9 = 1.
Dalla prima equazione otteniamo 12 x(λ − 4) = 0. Se λ = 4 dalla seconda equazione
2
si ha y = 27 8
che è impossibile in quanto (27/8)
9
> 1, in contrasto con il vincolo.
Se x = 0 dalla terza equazione otteniamo y = ±3 da cui λ = ± 92 , e i 2 punti
critici vincolati P1 = (0, 3), P2 = (0, −3). Essendo f (P1 ) = 9, f (P2 ) = −9, si ha
m = min f |g=0 = −9, M = max f |g=0 = 9.
2o metodo. Parametrizzando il vincolo con
(
x = 2 cos t
y = 3 sin t

otteniamo la funzione di una variabile

h(t) = f (2 cos t, 3 sin t) = 4 cos2 t + 9 sin t = 12 sin 2t

da studiare nell’intervallo [0, 2π]. Uguagliando a zero la derivata prima

h0 (t) = −8 cos t sin t + 9 cos t = cos t(9 − 8 sin t)

otteniamo sin t = 9/8, che è impossibile, oppure cos t = 0, da cui t = π2 , 32 π. Non c’è
bisogno di verificare se questi punti stazionari sono punti di minimo o di massimo. È
sufficiente calcolare h( π2 ), h( 23 π), e confrontarli con i valori agli estremi h(0), h(2π).
(Infatti h è continua in [0, 2π], quindi assume massimo e minimo in tale intervallo;
essendo derivabile, h assume il massimo e il minimo nei punti critici interni o agli
estremi.) Essendo h(0) = h(2π) = 4, h( π2 ) = 9, h( 23 π) = −9, riotteniamo il risultato
precedente.

3. Trovare il massimo e il minimo assoluti di f (x, y) = xy con il vincolo |x| + |y| = 1.



Il vincolo g(x, y) = |x| + |y| − 1 = 0 rappresenta il quadrato di lato 2 con
diagonali lungo gli assi coordinati. I punti (1, 0), (0, 1), (−1, 0), (0, −1) sono punti
non regolari per il vincolo in quanto la funzione g(x, y) non è derivabile parzialmente
in tali punti. Ad esempio nel punto (1, 0) si ha

g(1, h) − g(1, 0) 1 + |h| − 1


lim± = lim± = ±1,
h→0 h h→0 h
e dunque non esiste la derivata parziale ∂y g(1, 0). In tali punti f vale 0. (Si verifica
facilmente, anche se non è richiesto, che tali punti non sono punti di massimo o

7
minimo vincolato per f (x, y) = xy; ad esempio avvicinandosi al punto (0, 1) lungo
il vincolo si vede che f > 0 se x → 0+ e f < 0 se x → 0− .)
Sui lati del quadrato possiamo procedere con i moltiplicatori. Ad esempio nel primo
quadrante x > 0, y > 0, otteniamo il sistema

y = λ

x=λ

x + y = 1,

da cui y = x, 2x = 1, x = y = 21 = λ e il punto critico P1 = ( 21 , 21 ). Analogamente


negli altri quadranti otteniamo i punti critici P2 = (− 12 , 12 ), P3 = (− 12 , − 21 ), P4 =
( 21 , − 12 ). Essendo f (P1 ) = f (P3 ) = 41 , f (P2 ) = f (P4 ) = − 41 , otteniamo m =
min f |g=0 = − 41 , M = max f |g=0 = 14 . Osserviamo infine che nei punti P1 , P2 , P3 , P4
le curve di livello f (x, y) = f (Pj ), cioè le iperboli xy = ± 41 , sono tangenti al vincolo,
cioè ai lati del quadrato.

4. Trovare i punti a minima e massima distanza dall’origine della circonferenza di


equazione (x − 2)2 + (y − 1)2 − 1 = 0.
Disegnando la circonferenza g(x, y) = (x − 2)2 + (y − 1)2 − 1 = 0 si vede facilmente
che i punti cercati P1 , P2 si ottengono intersecando la circonferenza con la retta
y = x/2, che congiunge l’origine con il centro C = (2, 1) ed è perpendicolare alla
circonferenza in tali punti. Si ottiene P1 = (2 − √25 , 1 − √15 ), P2 = (2 + √25 , 1 + √15 ).
Con i moltiplicatori si tratta di ottimizzare la funzione distanza dall’origine, o il
suo quadrato f (x, y) = x2 + y 2 , sul vincolo g(x, y) = 0. Si ottiene il sistema

2x = 2λ(x − 2)

2y = 2λ(y − 1)

(x − 2)2 + (y − 1)2 = 1.

Deve essere (x, y) 6= (2, 1). Ricavando allora λ dalla prima e dalla seconda equazione
e uguagliando si ha
x y
= =⇒ (y − 1)x = (x − 2)y =⇒ x = 2y,
x−2 y−1
e sostituendo nel vincolo si riottiene il risultato precedente. Notiamo che nei punti
P1 , P2 le curve di livello f (x, y) = f (P1,2 ), cioè le circonferenze x2 + y 2 = kP1,2 k2 ,
sono tangenti al vincolo.

5. Trovare il massimo e il minimo assoluti di f (x, y) = x2 y lungo la curva x4 + y 4 = 1.


Notiamo innanzitutto che l’insieme A = {(x, y) ∈ R2 : x4 + y 4 = 1} è chiuso e
limitato in R2 . Che sia limitato è chiaro in quanto deve essere |x| ≤ 1, |y| ≤ 1.
Che sia chiuso segue dal seguente risultato generale: se g : R2 → R è una funzione
continua, allora l’insieme {(x, y) : g(x, y) = 0} è un sottoinsieme chiuso di R2 .
(Analogamente sono chiusi gli insiemi {(x, y) : g(x, y) ≥ 0}, {(x, y) : g(x, y) ≤ 0},

8
mentre gli insiemi {(x, y) : g(x, y) < 0}, {(x, y) : g(x, y) > 0}, {(x, y) : g(x, y) 6=
0} sono aperti.) Nel nostro caso l’insieme A è l’insieme dei punti in cui si annulla
la funzione g(x, y) = x4 + y 4 − 1, che è continua da R2 a R. Si può dimostrare che
A è una curva chiusa contenuta nel quadrato di lato 2 centrato nell’origine.
Dal teorema di Weierstrass f |g=0 deve ammettere massimo e minimo. Scrivendo il
sistema ∇(f − λg) = 0, g = 0, otteniamo

3
2xy = 4λx

x2 = 4λy 3

 4
x + y 4 = 1.

Dalla prima equazione si ha x(y − 2λx2 ) = 0. Se x = 0 si ottiene y = ±1 dalla


terza equazione e λ = 0 dalla seconda. Inoltre f (0, ±1) = 0. Se invece y =
2λx2 , sostituendo la seconda equazione in quest’ultima si ottiene y = 8λ2 y 3 , cioè
y(1−8λ2 y 2 ) = 0. Se y = 0 dalla seconda equazione si ottiene che anche x = 0 e ciò è
y2
1
impossibile. Se invece 8λ2 y 2 = 1, cioè y 2 = 8λ1 2 si ha y 4 = 64λ e x4 = 4λ 1
2 = 32λ4 , da
q4 √4
cui sostituendo nel vincolo otteniamo λ4 = 64 3
=⇒ λ = ± 4 64 3
= ± 2√32 . Da questa
q q √ q
1 4 2 4 2 1 2 4 2 1
otteniamo y = ± √ 4
3
, x = ± 3
. Essendo f (± ,
3 43
√ ) = √
4
27
, f (± 3
,−√
4 ) =
3

2
−√
4
27
, otteniamo infine
√ √
2 2
m = min f |g=0 = −√
4
, M = max f |g=0 = √
4
.
27 27

6. Trovare il massimo e il minimo assoluti di f (x, y) = |x| + |y| sulla curva x2 + y 2 = 1.


Il modo più semplice di risolvere il problema è quello geometrico. Osserviamo che
se k >√0 le curve di livello di f , di equazione |x| + |y| = k, sono dei quadrati di
lato k 2 con le diagonali lungo gli assi coordinati (mentre se k = 0 si riducono
all’origine e se k < 0 sono l’insieme vuoto). È facile rendersi conto che il vincolo
S 1 = {(x, y) ∈ R2 : x2 +y 2√= 1} è contenuto nella regione compresa tra i 2 quadrati
|x| + |y| = 1 e |x| + |y| = 2,

S 1 ⊂ {(x, y) ∈ R2 : 1 ≤ |x| + |y| ≤ 2},

il secondo dei quali è tangente al vincolo nei punti (± √12 , ± √12 ), mentre il primo
è inscritto in S 1 e lo interseca nei punti (0, ±1), (±1, 0). È evidente√allora che
f |S 1 assume il valore √massimo M nei punti di S 1 tali che f (x, y) = 2, da cui
M = f (± √12 , ± √12 ) = 2, ed il valore minimo m nei punti di S 1 tali che f (x, y) = 1,
da cui m = f (0, ±1) = f (±1, 0) = 1.
Notiamo che, mentre nei punti di massimo le curve di livello di f sono tangenti
al vincolo, ciò non accade nei punti di minimo che sono punti non regolari per la
funzione f in quanto non esiste ∇f in tali punti (si veda l’esercizio 3).

9
p
7. Studiare gli estremi vincolati della funzione f (x, y) = 2x2 − xy + y 2 lungo la
retta x + y = 8.
Innanzitutto f è definita e continua su tutto R2 essendo

2x2 − xy + y 2 = 2(x − 41 y)2 + 78 y 2 ≥ 0, ∀(x, y) ∈ R2 .

In secondo luogo, se sostituiamo il vincolo y = 8 − x in f otteniamo la funzione di


una variabile √
h(x) = f (x, 8 − x) = 2 x2 − 6x + 16,
che è continua su R e soddisfa limx→±∞ h(x) = +∞. Ne segue che f |x+y=8 non
ha massimo mentre deve avere minimo m. Questo si può ottenere annullando la
derivata di h, oppure con i moltiplicatori di Lagrange risolvendo il sistema

√ 4x−y

 =λ
 2 2x2 −xy+y2

√ 2y−x =λ

 2 2x2 −xy+y 2

x + y = 8,

che dà 4x − y = 2y − x, cioè y = 53 x, da cui x = 3, y = 5, m = 2 7.
2
Geometricamente le curve di livello f (x, y) = k (k√ > 0), cioè 2x − xy + y 2 = k 2 ,
sono delle ellissi centrate nell’origine. Per k = 2 7 l’ellisse 2x2 − xy + y 2 = 28 è
tangente alla retta x + y = 8 nel punto (3, 5).

8. Sia y = f (x) l’equazione di una curva regolare e sia P = (a, b) un punto non ap-
partenente ad essa. Detto Q = (x, f (x)) un punto variabile sulla curva, dimostrare
che la retta P Q è perpendicolare alla curva se e solo se la distanza P Q è minima o
massima.
Possiamo procedere direttamente scrivendo la distanza
p
P Q = (x − a)2 + (f (x) − b)2 = h(x)

e annullando la derivata di h(x), oppure con i moltiplicatori cercando gli estremi


2
della funzione P Q o più semplicemente P Q , cioè

f (x, y) = (x − a)2 + (y − b)2 con il vincolo g(x, y) = y − f (x) = 0.

Otteniamo il sistema 
0
2(x − a) = −λf (x)

2(y − b) = λ

y = f (x).

Sostituendo la seconda e la terza equazione nella prima si ottiene l’equazione

x − a + f 0 (x) (f (x) − b) = 0

10
che fornisce l’ascissa del punto Q sulla curva y = f (x) a minima o massima distanza
da P . Da questa otteniamo che se f 0 (x) 6= 0, x 6= a,
f (x) − b 1
=− 0 ,
x−a f (x)
cioè il coefficiente angolare della retta P Q è proprio quello della retta normale alla
retta tangente alla curva y = f (x) nel punto Q. Se invece f 0 (x) = 0 si ha x = a,
cioè la retta P Q è verticale mentre la retta tangente a f (x) in Q = (a, f (a)) è
orizzontale. In ogni caso abbiamo che la distanza P Q è minima o massima se e solo
se la retta P Q è perpendicolare alla curva y = f (x) in Q.
9. Determinare il punto Q sulla retta y = mx + q alla minima distanza dal punto
P = (a, b). Dimostrare poi che la distanza di P dalla retta, cioè la distanza P Q, è
|ma + q − b|
d = PQ = √ .
1 + m2

Posto f (x) = mx + q nell’esercizio precedente otteniamo che l’ascissa del punto Q


è determinata dall’equazione
x − a + m(mx + q − b) = 0,
da cui si ottiene facilmente
a + (b − q)m ma + m2 b + q
 
Q= , .
1 + m2 1 + m2
2
Calcolando la distanza P Q otteniamo
2  2
ma + m2 b + q

2 a + (b − q)m
PQ = −a + −b
1 + m2 1 + m2
1  2 2

= (−m(ma + q − b)) + (ma + q − b)
(1 + m2 )2
(ma + q − b)2
= ,
1 + m2
da cui il risultato.
10. Trovare i punti a minima e massima distanza dall’origine della curva di equazione
x2 − xy + y 2 = 1.
Si tratta di ottimizzare la funzione f (x, y) = x2 + y 2 con il vincolo g(x, y) = x2 −
xy +y 2 −1 = 0. L’equazione g = 0 definisce un’ellisse centrata nell’origine. Essendo
questo un insieme chiuso e limitato in R2 , la funzione f |g=0 ammette massimo e
minimo. Procedendo con i moltiplicatori di Lagrange otteniamo il sistema

2x = λ(2x − y)

2y = λ(−x + 2y)

 2
x − xy + y 2 = 1.

11
Non può essere nè y = 2x nè x = 2y perchè in entrambi i casi si ottiene x = y = 0
che contrasta con il vincolo. Possiamo allora ricavare λ dalla prime due equazioni
e uguagliare, ottenendo
2x 2y
= =⇒ x2 = y 2 =⇒ x = ±y,
2x − y −x + 2y
e in corrispondenza λ = 2 o λ = 2/3. Sostituendo nel vincolo otteniamo i punti
critici vincolati P1 = (1, 1), P2 = (−1, −1) se y = x, e P3 = ( √13 , − √13 ), P4 =
(− √13 , √13 ) se y = −x. Calcolando la funzione f in questi punti si ha

f (P1 ) = f (P2 ) = 2, f (P3 ) = f (P4 ) = 32 .


p √
Quindi la minima (risp. massima) distanza dall’origine è 2/3 (risp. 2). In
effetti si può dimostrare√che g =p0 definisce un’ellisse centrata nell’origine con assi
y = ±x e semiassi a = 2, b = 2/3. Questa ellisse è tangente alla circonferenza
x2 + y 2 = 2 (risp. x2 + y 2 = 2/3) nei punti P1 , P2 (risp. P3 , P4 ).

11. Determinare il minimo di f (x, y) = x2 + y 2 con il vincolo (x − 1)3 − y 2 = 0.


L’insieme A definito dal vincolo g(x, y) = (x − 1)3 − y 2 = 0 è l’unione dei grafici
delle due funzioni y = ±(x − 1)3/2 . Poichè f (x, y) è la distanza al quadrato di
(x, y) dall’origine, è chiaro (disegnando i due grafici) che f ha minimo assoluto
su A nel punto P = (1, 0) ed il minimo vale 1. Notiamo che il punto P è un
punto non regolare per il vincolo perchè il gradiente ∇g(x, y) = (3(x − 1)2 , −2y) si
annulla precisamente per x = 1, y = 0. Procedendo con i moltiplicatori otteniamo
il sistema 
2
2x = 3λ(x − 1)

2y = −2λy

 2
y = (x − 1)3
che non è soddisfatto nel punto (1, 0) qualunque sia λ in R. Dunque tale punto di
minimo non può essere determinato in questo modo. Si verifica facilmente che il
sistema non ha soluzioni.

12. Studiare gli estremi vincolati della funzione f (x, y) = −x log x−y log y con il vincolo
x + y = 1.
L’insieme sul quale dobbiamo considerare la funzione è

A = {(x, y) ∈ R2 : g(x, y) = x + y − 1 = 0, x > 0, y > 0}.

Si tratta del segmento di estremi (1, 0), (0, 1) sulla retta x + y = 1 (estremi esclusi).
Sostituendo y = 1 − x in f (x, y) otteniamo la funzione di 1 variabile

h(x) = f (x, 1 − x) = −x log x − (1 − x) log(1 − x)

nell’intervallo 0 < x < 1. Notiamo che limx→0+ h(x) = limx→1− h(x) = 0. Inoltre
poichè x e 1 − x sono sempre compresi tra 0 e 1 e log t < 0 per 0 < t < 1, si

12
ha che h(x) è sempre positiva. Ne segue che h deve avere massimo. Per trovare
tale massimo possiamo annullare la derivata h0 , oppure possiamo procedere con i
moltiplicatori ottenendo il sistema

− log x − 1 = λ

− log y − 1 = λ

x + y = 1.

Le soluzione è x = y = 1/2, λ = log 2 − 1, e il massimo è M = max f |g=0 = log 2.


Non esiste invece il minimo di f , mentre l’estremo inferiore è zero.
13. Studiare gli estremi vincolati della funzione f (x, y) = x + y con il vincolo xy = 1,
x > 0, y > 0.
Possiamo sostituire il vincolo y = 1/x e studiare la funzione di una variabile h(x) =
x + 1/x nell’intervallo (0, 1), oppure procedendo con i moltiplicatori otteniamo il
sistema 
1 = λx

1 = λy

xy = 1,

la cui unica soluzione è x = y = λ = 1, con f (1, 1) = 2. Studiando la funzione h(x)


si vede subito che (1, 1) è punto di minimo vincolato (assoluto). Non esiste invece
il massimo di f lungo il vincolo. Infatti il vincolo è il ramo d’iperbole y = 1/x con
x > 0. Se x → 0+ , y → +∞, e analogamente se y → 0+ , x → +∞. Quindi la
funzione f (x, y) = x + y non è limitata superiormente lungo il vincolo.
x2 y2
14. Tra tutti i rettangoli inscritti nell’ellisse di equazione a2
+ b2
= 1 trovare quello di
perimetro massimo.
Detti x, y i semilati del rettangolo, si tratta di massimizzare la funzione f (x, y) =
2 2
4(x + y) con il vincolo xa2 + yb2 = 1, x ≥ 0, y ≥ 0. Procedendo con i moltiplicatori
otteniamo il sistema 
2
4 = 2λx/a

4 = 2λy/b2
 x2 y 2

a2
+ b2 = 1.
x y b2
Deve essere λ 6= 0 e dalle prime 2 equazioni si ha a2
= b2
, cioè y = a2
x. Sostituendo
nel vincolo si ottiene facilmente
a2 b2
x= √ , y=√ ,
a2 + b 2 a2 + b 2
√ √
e infine λ = 2 a2 + b2 . Il perimetro massimo è pmax = 4 a2 + b2 . Il rettangolo
cercato è quello che ha per semilati le coordinate del punto di intersezione dell’ellisse
2
con la retta y = ab 2 x (nel primo quadrante). In particolare se a = b otteniamo che
il rettangolo
√ di perimetro massimo inscritto in un cerchio di raggio a è quello con
x = y = a/ 2, cioè il quadrato.

13
È evidente geometricamente che l’estremo vincolato ottenuto corrisponde ad un
massimo. Il rettangolo di perimetro minimo è quello degenere in cui uno dei due
lati è zero, cioè x = 0, y = b (se a > b), oppure y = 0, x = a (se a < b), con
perimetro pmin = min{4b, 4a}.

15. Tra tutte le ellissi di semiassi x, y tali che x2 +y 2 = 5 trovare quella di area massima.
Dobbiamo massimizzare la funzione f (x, y) = πxy con il vincolo x2 + y 2 = 5, x > 0,
y > 0. Con i moltiplicatori otteniamo

πy = 2λx

πx = 2λy

 2
x + y 2 = 5.

= x/y da cui x2 = y 2 e x = ±y.


Dovendo essere λ 6= 0 si ha y/x p p Sostituendo
nel vincolo otteniamo x = y = 5/2, cioè il cerchio di raggio 5/2, con area
Amax = 5π/2. È evidente che√ la “minima” area Amin
√ = 0 corrisponde al caso
degenere in cui x → 0 e y → 5 oppure y → 0 e x → 5.

16. Tra tutti i cilindri inscritti nella sfera di raggio R trovare quello di volume massimo.
Detto x il raggio del cilindro e y la semialtezza, dobbiamo trovare il massimo della
funzione f (x, y) = (πx2 ) · (2y) = 2πyx2 con il vincolo x2 + y 2 = R2 , x > 0, y > 0.
Con i moltiplicatori otteniamo il sistema

4πxy = 2λx

2πx2 = 2λy

 2
x + y 2 = R2 .

Non può essere λ = 0 nè x = 0 nè y = 0. Eliminando


√ λ dalle prime 2 equazioni
2 2 2
q da cui 2y = x e y = x/ 2. Sostituendo nel vincolo
si ottiene 2πy = πx /y,
2 √1 R.
otteniamo infine x = 3
R, y= 3
Notiamo che il volume massimo è Vmax =
4πR 3 √
√ , cioè il volume della sfera diviso per 3.
3 3

17. Tra tutti i cilindri (circolari retti) di superficie totale S fissata (incluse le due facce
circolari) trovare quello di volume massimo.
Detto x il raggio e y l’altezza del cilindro, dobbiamo massimizzare la funzione
f (x, y) = πx2 y con il vincolo g(x, y) = 2πxy + 2πx2 = S, x > 0, y > 0. Procedendo
con i moltiplicatori otteniamo il sistema

2πxy = λ(2πy + 4πx)

πx2 = 2πλx

2πxy + 2πx2 = S.

14
Dalla seconda equazione si ottiene λ = x/2 che sostituita nella prima dà y = 2x. Il
volume massimo si ottiene dunque quando l’altezza del cilindro è uguale al diametro.
Sostituendo nel vincolo otteniamo
r  3/2
S S
x= , Vmax = 2π .
6π 6π

18. Trovare il massimo e il minimo assoluti della funzione f (x, y, z) = xy + xz + yz


sulla sfera x2 + y 2 + z 2 = 1.
Notiamo che la sfera S 2 = {(x, y, z) ∈ R3 : x2 + y 2 + z 2 = 1} è un sottoinsieme
chiuso e limitato di R3 . Poichè la funzione f è continua, la restrizione f |S 2 ammette
massimo e minimo per il teorema di Weierstrass. Applichiamo il metodo dei molti-
plicatori di Lagrange. Detta g(x, y, z) = x2 + y 2 + z 2 − 1, il sistema ∇(f − λg) = 0,
g = 0 diventa 

 y + z = 2λx

x + z = 2λy


 x + y = 2λz
 2
x + y 2 + z 2 = 1.
Notiamo che non può essere λ = 0 perchè in tal caso si ha 0 = y +z = x+z = x+y,
da cui si ottiene facilmente x = y = z = 0, che non soddisfa il vincolo. Ricavando
1
allora x dalla prima equazione, x = 2λ (y + z), e sostituendo nella seconda e nella
terza otteniamo il sistema
(
(4λ2 − 1)y = (2λ + 1)z
(3)
(2λ + 1)y = (4λ2 − 1)z.

Si presentano 2 casi, cioè λ = −1/2 e λ 6= −1/2. 1) Se λ = −1/2 entrambe le


1
equazioni in (3) sono soddisfatte. Inoltre da x = 2λ (y + z) si ottiene x = −(y + z),
cioè x + y + z = 0. In questo caso tutti i punti (x, y, z) tali che
(
x+y+z =0
x2 + y 2 + z 2 = 1
soddisfano il sistema e sono punti critici vincolati. Notiamo che tali punti sono
l’intersezione della sfera con il piano x+y +z = 0 e rappresentano una circonferenza
in tale piano, la cui proiezione nel piano xy è l’ellisse di equazione 2x2 +2y 2 +2xy = 1
(come si vede subito eliminando z). In questi punti la funzione vale identicamente
−1/2, infatti si ha
f (x, y, z) = xy + xz + yz = 12 (x + y + z)2 − 21 (x2 + y 2 + z 2 )
e dunque f (x, y, z)|x+y+z=0, x2 +y2 +z2 =1 = −1/2.
2) Se λ 6= −1/2 il sistema (3) diventa (dividendo per (2λ + 1))
(
(2λ − 1)y = z
y = (2λ − 1)z.

15
Sostituendo la prima equazione nella seconda si ottiene y = (2λ − 1)2 y, da cui
4λ(λ − 1)y = 0. Non può essere nè λ = 0 (come già osservato) nè y = 0 (in
entrambi i casi si ottiene x = y = z = 0). Dunque necessariamente λ = 1, e si
ottiene facilmente che y = z = x = ± √13 . Quindi in questo caso otteniamo i 2 punti
critici vincolati P1,2 = ±( √13 , √13 , √13 ). Essendo f (P1 ) = f (P2 ) = 1 concludiamo che

M = max f |g=0 = 1, m = min f |g=0 = −1/2.

Il massimo è assunto nei punti P1 , P2 ; il minimo è assunto in tutti i punti della sfera
tali che x + y + z = 0. Un modo più veloce per risolvere l’esercizio è indicato negli
esercizi 29 e 32.

19. Trovare il massimo e il minimo assoluti della funzione f (x, y, z) = (x + y + z)2 con
il vincolo x2 + 2y 2 + 3z 2 = 1.
3 2 2 2
L’insieme A = {(x, y, z)
√ ∈ R : g(x, √ y, z) = x + 2y + 3z − 1 = 0} è un ellissoide di
semiassi a = 1, b = 1/ 2, c = 1/ 3. Questo è un sottoinsieme chiuso e limitato di
R3 . Essendo f continua, la restrizione f |A ammette massimo e minimo. Procedendo
con il metodo dei moltiplicatori di Lagrange otteniamo il sistema


 2(x + y + z) = 2λx

2(x + y + z) = 4λy
2(x + y + z) = 6λx


 2
x + 2y 2 + 3z 2 = 1.

Dalle prime 3 equazioni otteniamo λx = 2λy = 3λz. Se λ = 0 si ottiene x+y+z = 0.


Essendo f (x, y, z) = (x + y + z)2 ≥ 0, tutti i punti (x, y, z) tali che
(
x+y+z =0
x2 + 2y 2 + 3z 2 = 1

sono punti critici vincolati di minimo assoluto per f , con m = min f |A = 0. Se


invece λ 6= 0 otteniamo x = 2y = 3z. Sostituendo nel vincolo otteniamo i 2 punti
critici vincolati  q q q 
2 3 2 2
P1 = 3 ,
33 2 33
, 33
, P2 = −P1 ,

da cui il massimo assoluto


q 2
2 3 11
M = max f |A = f (P1 ) = f (P2 ) = 33
(3 + + 1)
2
= 6
.

Un altro modo per risolvere l’esercizio è proposto nell’esercizio 35.

20. Tra tutti i parallelepipedi (rettangoli) di volume assegnato V determinare quello di


superficie minima.

16
Detti x, y, z i 3 lati, dobbiamo minimizzare la funzione f (x, y, z) = 2(xy + xz + yz)
con il vincolo g(x, y, z) = xyz − V = 0, x > 0, y > 0, z > 0. Il sistema ∇(f − λg) =
0, g = 0 diventa 

 2(y + z) = λyz

2(x + z) = λxz
2(x + y) = λxy



xyz = V.
y+z x+z x+y
Eliminando λ otteniamo yz
= xz
= .
Da queste si ottiene facilmente x = y =
xy √
z, cioè il parallelepipedo di superficie minima è il cubo di spigolo 3 V . È evidente
geometricamente che il punto critico x = y = z corrisponde a un minimo e non
certo a un massimo. Infatti mantenendo V = xyz costante possiamo aumentare
indefinitamente la funzione f contraendo uno dei lati a zero e facendo tendere un
altro lato a +∞. In altri termini, la funzione f non è limitata superiormente sul
vincolo g = 0. Per esempio prendendo (x, y, z) = (V, 1ε , ε) e facendo tendere ε → 0+ ,
si ha che

xyz = V, z → 0+ , y → +∞, f (x, y, z) = 2( Vε + V ε + 1) → +∞.

21. Trovare i punti a minima e massima distanza dall’origine dell’ellisse ottenuto


2 2
tagliando l’ellissoide x4 + y4 + z 2 = 1 con il piano x + y + z = 1.
Possiamo ottimizzare la funzione f (x, y, z) = x2 + y 2 + z 2 con i 2 vincoli g(x, y, z) =
2
x2
4
+ y4 + z 2 − 1 = 0, h(x, y, z) = x + y + z − 1 = 0. Con il metodo dei moltiplicatori
dobbiamo risolvere il sistema


 2x = λ2 x + µ
 
λ
∇(f − λg − µh) = 0 2y = 2 y + µ


 
g=0 cioè 2z = 2λz + µ
2
x2
+ y4 + z 2 = 1
 
h=0
 



 4
x + y + z = 1.

Eliminando µ dalle prime 2 equazioni otteniamo (4 − λ)x = (4 − λ)y, e dunque


λ = 4 oppure x = y. Se λ = 4 allora risostituendo nella prima o nella seconda
equazione si ottiene µ = 0, e dalla terza segue z = 0. Sostituendo z = 0 nei 2
vincoli si ottiene il sistema (
x2 + y 2 = 4
x+y =1
√ √
1± 7 1∓ 7
da cui x = 2
, y= 2
. Otteniamo cosı̀ i 2 punti critici vincolati
√ √ √ √
P1 = ( 1+2 7 , 1−2 7 , 0), P2 = ( 1−2 7 , 1+2 7 , 0),

con f (P1 ) = f (P2 ) = 4. Se invece x = y allora (dal secondo vincolo) z = 1 − 2x e


sostituendo nel primo vincolo otteniamo 12 x2 + (1 − 2x)2 = 1 da cui x = 0, 89 . Se

17
8
x = y = 0 allora z = 1 e µ = 0, λ = 1. Se x = y = 9
allora z = − 97 , λ = 35 , µ = 28
27
.
Otteniamo cosı̀ i 2 punti critici vincolati

P3 = (0, 0, 1), P4 = ( 98 , 89 , − 97 ),

con f (P3 ) = 1, f (P4 ) = 59 < 4. Concludiamo che il punto a minima distanza è P3


27 √
con dmin = 1, mentre i punti P1 e P2 sono a massima distanza con dmax = 4 = 2.
Il punto P4 non è invece nè punto di massimo nè di minimo vincolato. Il suo
significato è di essere il punto a minima quota z = − 79 lungo l’ellisse, come si vede
estremando la funzione f (x, y, z) = z con gli stessi vincoli. Procedendo come sopra
si ottengono i 2 punti critici P3 e P4 con zmax = f (P3 ) = 1 e zmin = f (P4 ) = − 97 .

22. Tra tutti i triangoli di perimetro assegnato 2p trovare quello di area massima.
Siano x, y, zp
i tre lati del triangolo. Per la formula di Erone, dobbiamo massimizzare
la funzione p(p − x)(p − y)(p − z) o il suo quadrato

f (x, y, z) = p(p − x)(p − y)(p − z)

con i vincoli

x + y + z = 2p, 0 < x < p, 0 < y < p, 0 < z < p.

(Il caso in cui un lato è uguale a p è un caso degenere in cui i 3 vertici del triangolo
sono allineati e l’area è zero.) Procedendo con i moltiplicatori otteniamo il sistema


−p(p − y)(p − z) = λ

−p(p − x)(p − z) = λ
−p(p − x)(p − y) = λ



x + y + z = 2p.

Dalle prime 2 equazioni si ha (p − y)(p − z) = (p − x)(p − z), da cui essendo p 6= z


otteniamo x = y. Analogamente dalla seconda e terza equazione si ottiene y = z.
In definitiva il triangolo di area massima è√ quello√con x = y = z, cioè il triangolo
equilatero di lato l = 2p/3 e area Amax = 43 l2 = 93 p2 .

23. a) Trovare il minimo della funzione f (x1 , x2 , . . . , xn ) = x1 + x2 + · · · + xn con il


vincolo x1 · x2 · · · xn = k (k fissato > 0) e xj > 0 ∀j.
b) Dedurre la seguente disuguaglianza tra media geometrica e aritmetica: dati n
numeri positivi x1 , x2 , . . . , xn vale
√ x1 + x2 + · · · + xn
n
x1 x2 · · · xn ≤
n
con uguaglianza se e solo se x1 = x2 = · · · = xn .

18
a) Posto g(x1 , . . . , xn ) = x1 · x2 · · · xn − k, il sistema ∇(f − λg) = 0, g = 0 diventa


 1 = λx2 · x3 · · · xn

1 = λx1 · x3 · · · xn



..
 .
1 = λx1 · x2 · · · xn−1





x1 · x2 · · · xn = k.

Moltiplicando la prima equazione per x1 , la seconda per x2 , la n-esima per xn e


tenendo conto del vincolo otteniamo che

x1 = x2 = · · · = xn = λk.

n
Risostituendo nel vincolo abbiamo che (λk)n = k, λk = k da cui il punto critico

n
x1 = x2 = · · · = xn = k.

È evidente che si tratta di un minimo e non di un massimo. Infatti prendendo ad


esempio x1 = kA e x2 = x3 = · · · = xn = n−11√A e facendo tendere A → +∞ il
prodotto x1 · x2 · · · xn è costante uguale a k mentre
n−1
f (x1 , x2 , . . . , xn ) = kA + √ → +∞.
n−1
A
Dunque la funzione f non è limitata superiormente sul vincolo x1 · x2 · · · xn = k.
(Si veda anche l’esercizio 13 per il caso n=2.)

b) Essendo il punto x1 = x2 = · · · = xn = n k un punto di minimo, si deve avere
che per ogni n-upla di numeri positivi x1 , . . . , xn , detto k il loro prodotto,

n

n

n
f (x1 , x2 , . . . , xn ) ≥ f ( k, k, . . . k),

cioè √
n √
x1 + x2 + · · · + xn ≥ n k=n n
x1 · x2 · · · xn .
Dividendo per n si ottiene la disuguaglianza voluta. L’uguaglianza vale solo nel
punto critico, cioè se e solo se x1 = x2 = · · · = xn .

24. Calcolare il massimo e il minimo assoluti della funzione f (x, y) = |x| + |y|
sull’insieme A = {(x, y) ∈ R2 : x2 + y 2 ≤ 1}. (Si ricordi l’es. 6.)
Poichè f è continua e A è compatto (chiuso e limitato), f ammette massimo M
e minimo m su A per il teorema di Weierstrass. Nell’esercizio 6 dei risolti abbi-
amo calcolato gli estremi √ vincolati di f sulla frontiera ∂A di A, con il risultato
min f |∂A = 1, max f |∂A = 2. Consideriamo ora gli estremi liberi di f nell’interno
di A. Per il teorema di Fermat i punti di estremo (massimo o minimo) in cui
f è differenziabile sono da ricercare tra i punti critici di f , cioè i punti in cui si
annulla il gradiente. I punti in cui f non è differenziabile vanno studiati a parte.

19
Nel nostro caso f è differenziabile in tutti i punti che non stanno sugli assi coor-
dinati, mentre @ ∂y f (x̄, 0), @ ∂x f (0, ȳ), ∀x̄, ȳ, come si verifica subito. Nell’origine
non esistono nè ∂x f (0, 0) nè ∂y f (0, 0). Nei punti di differenziabilità il gradiente
∇f non si annulla mai. Per esempio nel primo quadrante x > 0, y > 0, si ha
f (x, y) = x + y e ∇f (x, y) = (1, 1). Lungo gli assi si ha f (x̄, 0) = |x̄|, f (0, ȳ) = |ȳ|,
quindi nell’insieme A, 0 ≤ f (x̄, 0) ≤ 1, 0 ≤ f (0, ȳ) ≤ 1, e nell’origine f (0, 0) = 0.
Essendo f (x, y) ≥ 0 ∀(x, y), è evidente che (0, 0) è un punto di minimo assoluto √
per f . Concludiamo che f assume il massimo assoluto sulla frontiera, M = 2, e
il minimo assoluto nell’origine, m = 0.
2 −y 2
25. Trovare il massimo e il minimo assoluti della funzione f (x, y) = ex sugli insiemi
a) A = {(x, y) ∈ R2 : x2 + y 2 ≤ 1};
b) B = {(x, y) ∈ R2 : |x| ≤ 1, |y| ≤ 1}.
a) Dobbiamo estremizzare f sul cerchio unitario. Distinguiamo lo studio dei punti
interni da quello della frontiera ∂A = {(x, y) ∈ R2 : x2 + y 2 = 1}.

• Interno. Calcolando il gradiente di f e uguagliando a zero otteniamo


2 −y 2
∇f (x, y) = ex (2x, −2y) = (0, 0) ⇐⇒ x = y = 0,

con f (0, 0) = 1. (Non è richiesto di verificare se si tratta di un punto di


massimo, minimo o sella. Comunque calcolando il determinante hessiano si
2 0
trova Hf (0, 0) = = −4 < 0 e quindi l’origine è un punto di sella.)
0 −2
• Frontiera. Procedendo con i moltiplicatori abbiamo che gli eventuali punti di
estremo vincolato sono soluzione del sistema
 2 2
x −y
e
 2x = 2λx
2 2
−ex −y 2y = 2λy

 2
x + y 2 = 1.

Notiamo che se vale contemporaneamente x 6= 0 e y 6= 0 allora eliminando


2 2 2 2
λ tra le prime 2 equazioni si ottiene λ = ex −y = −ex −y che è impossibile.
Quindi o x = 0 o y = 0. Se x = 0 dal vincolo si ottiene y = ±1, e dalla seconda
equazione λ = −e−1 . Quindi i 2 punti (0, ±1) sono punti critici vincolati con
f (0, ±1) = e−1 = 1e . Se invece y = 0 otteniamo x = ±1, λ = e, f (±1, 0) = e.

Concludiamo che M = max f |A = e, m = min f |A = 1e .


b) Notiamo che l’insieme B è il quadrato −1 ≤ x ≤ 1, −1 ≤ y ≤ 1, che è chiuso e
limitato, quindi f essendo continua deve avere massimo e minimo su B.

• Interno. Procedendo come sopra l’unico punto critico interno è l’origine con
f (0, 0) = 1.

20
• Frontiera. ∂B è l’unione di 4 curve regolari (i 4 lati) ma i vertici P1 = (1, 1),
P2 = (−1, 1), P3 = (−1, −1), P4 = (1, −1) sono punti non regolari e vanno
considerati a parte. In tali punti si ha f (P1 ) = f (P2 ) = f (P3 ) = f (P4 ) = 1.
Sui lati del quadrato si ha:
2
(a) per x = 1 e −1 < y < 1 si ha f (1, y) = e1−y , che presenta un massimo in
y = 0, con f (1, 0) = e;
2
(b) per x = −1 e −1 < y < 1 si ha f (−1, y) = e1−y , e otteniamo la stessa
conclusione;
2
(c) per y = ±1 e −1 < x < 1 si ha f (x, ±1) = ex −1 , che ha un minimo in
x = 0, con f (0, ±1) = e−1 .

Confrontando tutti i valori di f concludiamo che M = max f |B = e, m = min f |B =


1
e
, esattamente come sull’insieme A.

26. Trovare il massimo e il minimo assoluti di f (x, y) = x2 y + xy 2 − xy nel triangolo


T = {(x, y) ∈ R2 : x + y ≤ 1, x ≥ 0, y ≥ 0}.

• Interno. I punti critici interni sono determinati da

∇f (x, y) = (2xy + y 2 − y, x2 + 2xy − x) = (0, 0),

cioè sono le soluzioni del sistema


(
y(2x + y − 1) = 0
x(x + 2y − 1) = 0.

Per la legge di annullamento del prodotto otteniamo l’unione di 4 sistemi


( ( ( (
y=0 y=0 2x + y − 1 = 0 2x + y − 1 = 0
∪ ∪ ∪
x=0 x + 2y − 1 = 0 x=0 x + 2y − 1 = 0.

La soluzione è data dai 4 punti (0, 0), (1, 0), (0, 1), ( 31 , 13 ). I primi 3 sono sulla
frontiera ∂T (sono proprio i vertici del triangolo) e sono punti non regolari
di ∂T che vanno comunque considerati a parte. In tali punti si ha f (0, 0) =
f (1, 0) = f (0, 1) = 0. Il punto ( 13 , 31 ) è invece interno a T e si ha f ( 13 , 13 ) = − 27
1
.
• Frontiera. Abbiamo già visto i vertici, consideriamo ora i lati del triangolo.
(a) y = 0, 0 < x < 1, f (x, 0) = 0, ∀x;
(b) x = 0, 0 < y < 1, f (0, y) = 0, ∀y;
(c) y = 1 − x, 0 < x < 1, f (x, 1 − x) = x2 (1 − x) + x(1 − x)2 − x(1 − x) =
(1 − x)(x2 + x − x2 − x) = 0, ∀x.
1
Concludiamo che M = max f = 0, m = min f = − 27 .

27. Trovare il massimo e il minimo assoluti di f (x, y) = x2 + y 2 − 3y + xy nel cerchio


A = {(x, y) ∈ R2 : x2 + y 2 ≤ 9}.

21
• Interno. I punti critici interni sono determinati da

∇f (x, y) = (2x + y, 2y − 3 + x) = (0, 0),

da cui il sistema
( (
2x + y = 0 y=2
⇐⇒
2y − 3 + x = 0 x = −1.

C’è quindi un solo punto critico interno P1 = (−1, 2) e si calcola f (P1 ) = −3.
Pur non essendo
 richiesto,
 calcolando la matrice hessiana in P1 si ottiene
2 1
Hf (−1, 2) = , e si deduce che P1 è un punto di minimo relativo.
1 2
• Frontiera. Procedendo con il metodo dei moltiplicatori di Lagrange, dobbiamo
risolvere il sistema 
2x + y = 2λx

2y − 3 + x = 2λy

 2
x + y 2 = 9.
Se x = 0 dalla prima equazione si ha y = 0 che è impossibile. Se y = 0 dalla
seconda equazione si ha x = 3 ed il punto (3, 0) risolve anche il vincolo. Inoltre
dalla prima si ottiene λ = 1. Se invece y 6= 0 e x 6= 0, eliminando λ tra le
prime 2 equazioni otteniamo
2x + y 2y − 3 + x
2λ = = ,
x y

da cui 2xy + y 2 = 2xy − 3x + x2 , cioè y 2 = −3x + x2 . Sostituendo nel vincolo


otteniamo l’equazione 2x2 − 3x − 9 = 0, da cui x = 3, −3/2. Se x = 3 si
ha y = √0, cioè si riottiene la soluzione (3, 0). Se invece x = −3/2 otteniamo
y = ±3 3/2. In definitiva, √ otteniamo 3 punti √ critici vincolati sulla frontiera,
P2 = (3, 0), P3 = (− 32 , 32 3), P4 = (− 23 , − 23 3).
√ √
Si calcola f (P2 ) = 9, f (P3 ) = 9 − 27 3 < 9, f (P 4 ) = 9 + 27
3 > 9. Notiamo
27
√ 4 4
inoltre che 9 − 4 3 > −3, cioè f (P3 ) > f (P1 ), come si verifica facilmente (anche
senza calcolatrice!). Confrontando
√ tutti i valori di f nei punti critici concludiamo
27
che M = max f = 9 + 4 3, m = min f = −3.

Esercizi di approfondimento

28. a) Scrivendo il sistema ∇(f − λg)(x, y, z) = 0 otteniamo




y + z = a2 x

x + z = 2λb2
y (4)
 2λ
x + y = c2 z,

22
cioè 

− a2 x + y + z = 0

x − 2λ
b2
y+z =0

x + y − 2λ z = 0,

c2

o ancora, in forma matriciale,


   2λ  
x − a2 1 1 x

A(λ) y
  =  1 − b2 1   y  = 0.

z 1 1 − c2 z

Affinchè vi siano soluzioni non identicamente nulle (questa condizione è richiesta


per soddisfare il vincolo) è necessario e sufficiente che il determinante della matrice
A(λ) si annulli. Un semplice calcolo ci dà

− 2 1 1
= − 2
a 2λ
3 2 2 2 2 2 2
 2 2 2

1 − 2 1 4λ − a b + a c + b c λ − a b c
b
a2 b 2 c 2
1 1 − 2λ c2

2
= − 2 2 2 p(λ).
abc

b) Moltiplicando la prima equazione del sistema (4) per x, la seconda per y, la terza
per z e sommandole, otteniamo che se (x, y, z, λ) risolve il sistema (1) allora
 2
y2 z2

x
x(y + z) + y(x + z) + z(x + y) = 2λ + 2 + 2 = 2λ,
a2 b c
cioè 2(xy + xz + yz) = 2λ, cioè f (x, y, z) = λ.
c) Si ha limλ→±∞ p(λ) = ±∞. La derivata è

p0 (λ) = 12λ2 − a2 b2 + a2 c2 + b2 c2 ,


dunque r
a2 b2 + a2 c2 + b2 c2
p0 (λ) = 0 ⇐⇒ λ = λ± = ± .
12
La funzione p(λ) è crescente su (−∞, λ− ) e su (λ+ , +∞), decrescente su (λ− , λ+ ),
e ha un massimo per λ = λ− e un minimo per λ = λ+ . Poichè p(0) < 0, è chiaro
che p(λ+ ) < 0 e p(λ) ha sicuramente una radice positiva λ3 > 0. Per vedere se vi
sono radici negative bisogna studiare il segno del massimo p(λ− ). Si ha
 2 2 3/2 3/2
a b + a2 c 2 + b 2 c 2 (a2 b2 + a2 c2 + b2 c2 )
p(λ− ) = −4 + √ − a2 b 2 c 2
12 12
 2 2 3/2
a b + a2 c 2 + b 2 c 2
= − a2 b 2 c 2 ≥ 0
3
a2 b2 + a2 c2 + b2 c2 2/3
⇐⇒ ≥ a2 b 2 c 2 = (abc)4/3 . (5)
3

23
Ora scrivendo la disuguaglianza ottenuta nell’esercizio 23 per n = 3 con x1 = a2 b2 ,
x2 = a2 c2 , x3 = b2 c2 , si ottiene
a2 b 2 + a2 c 2 + b 2 c 2 √
3
≥ a4 b4 c4 ,
3
che è precisamente la (5). Ne segue che p(λ− ) ≥ 0, con uguaglianza se e solo se
x1 = x2 = x3 cioè a = b = c. Dunque p(λ) ha sempre 2 radici negative λ1 < λ2 < 0
tranne che per a = b = c, in cui ha una sola radice negativa (di molteplicità 2, che
è proprio λ = λ− , cioè il punto di massimo in cui p(λ) si annulla).
d) Dai punti a), b) e c) segue che il sistema (1) ha soluzione se e solo se λ =
λ1 , λ2 , λ3 . Inoltre se (xj , yj , zj ) è una qualsiasi soluzione corrispondente a λj , si ha
che λj = f (xj , yj , zj ) (j = 1, 2, 3). Per il teorema di Weierstrass esistono il minimo
m e il massimo M di f |g=0 (l’ellissoide essendo chiuso e limitato in R3 ). Per il
teorema dei moltiplicatori di Lagrange, detto (xm , ym , zm ) un punto di minimo e
(xM , yM , zM ) un punto di massimo di f vincolati a g = 0, esistono λm e λM tali che il
sistema (1) è soddisfatto da (xm , ym , zm , λm ) e da (xM , yM , zM , λM ). Essendo λm =
f (xm , ym , zm ) = m e λM = f (xM , yM , zM ) = M , segue che m, M ∈ {λ1 , λ2 , λ3 }.
Poichè λj = f (xj , yj , zj ), la più piccola radice λ1 di p(λ) deve necessariamente essere
m, la più grande λ3 deve essere M .

29. a) Per a = b = c il polinomio p(λ) dell’esercizio precedente diventa

p(λ) = 4λ3 − 3a4 λ − a6 .

Si verifica immediatamente che p(a2 ) = 0. Scomponendo con Ruffini otteniamo

p(λ) = (λ − a2 )(2λ + a2 )2 ,
2
da cui le radici λ1 = λ2 = − a2 , λ3 = a2 .
b) Per a = b = c e λ = λ3 = a2 il sistema (1) diventa (si veda la (4)):


 y + z = 2x

x + z = 2y


 x + y = 2x
 2
x + y 2 + z 2 = a2 .

Dalla prima equazione ricaviamo x = 21 (y + z) e sostituendo nella seconda e terza


equazione otteniamo
( (
1
2
(y + z) + z = 2y − 32 y + 32 z = 0
1
⇐⇒ 3
2
(y + z) + y = 2z 2
y − 32 z = 0,

da cui y = z, e dunque x = y = z. Sostituendo nel vincolo si ottiene x = y = z =


± √a3 , e quindi abbiamo i 2 punti critici vincolati ±v3 = ±( √a3 , √a3 , √a3 ), che sono
punti di massimo assoluto f (±v3 ) = a2 = λ3 = M .

24
Per a = b = c e λ = λ1 = −a2 /2 il sistema (1) diventa


 y + z = −x (

x + z = −y x+y+z =0
cioè


 x + y = −x x 2 + y 2 + z 2 = a2 .
 2
x + y 2 + z 2 = a2

In questo caso tutti i vettori v = (x, y, z) tali che kvk = a e x + y + z = 0


sono soluzione. Tutti questi sono punti di minimo assoluto m = λ1 = −a2 /2.
In particolare per a = 1 riotteniamo i risultati dell’es. 18 dei risolti (M = 1,
m = −1/2).

30. Per a = b 6= c il polinomio p(λ) dell’esercizio 28 diventa

p(λ) = 4λ3 − a4 + 2a2 c2 λ − a4 c2 .




2
È facile verificare che p(− a2 ) = 0. Scomponendo con Ruffini otteniamo
 2

p(λ) = λ + a2 4λ2 − 2a2 λ − 2a2 c2 .


Uguagliando a zero il trinomio di secondo grado si ottiene



a2 a2 ± a a2 + 8c2
p(λ) = 0 ⇐⇒ λ = − , .
2 4
Dall’esercizio 28 segue che il massimo M di f |g=0 è la più grande radice di p(λ),
dunque √
a2 + a a2 + 8c2
M= .
4
Analogamente il minimo m di f |g=0 è la più piccola√
radice di p(λ). Per vedere chi è
a2 a2 −a a2 +8c2
la più piccola tra le 2 radici negative − 2 e 4
studiamo la disuguaglianza

a2 −a a2 +8c2 2
4
> − a2
2

⇐⇒ 3a > a a2 + 8c2

⇐⇒ 3a > a2 + 8c2
⇐⇒ 9a2 > a2 + 8c2
⇐⇒ a2 > c2 ⇐⇒ a > c.

Ne segue che ( 2
− a2 √
se a > c
m= a2 −a a2 +8c2
4
se a < c.
In particolare per a√ = b = 2, c = 1 riotteniamo i risultati dell’esercizio 12 dei
proposti (M = 1 + 3, m = −2).

25
31. a) Sia λ1 = max f |S n−1 e sia v1 ∈ S n−1 un punto di massimo, λ1 = f (v1 ). Per il
teorema dei moltiplicatori di Lagrange esiste λ ∈ R tale che

∇(f − λg)(v1 ) = 0.

Essendo
g(x) = x21 + x22 + · · · + x2n − 1,
è immediato che ∇g(x) = 2x. Per calcolare ∇f (x), scriviamo
n
X
f (x) = hx, Axi = Aij xi xj ,
i,j=1

e osserviamo che (
∂xi 1 se i = j
= δij =
∂xj 0 se i 6= j.
Ne segue che
n
∂f ∂ X
(x) = Aij xi xj
∂xk ∂xk i,j=1
n
X n
X
= Aij δik xj + Aij δjk xi
i,j=1 i,j=1
X n n
X
= Akj xj + Aik xi
j=1 i=1
Xn Xn
= Akj xj + Aki xi
j=1 i=1
Xn
=2 Akj xj = 2(Ax)k ,
j=1

da cui la formula ∇f (x) = 2Ax. Sostituendo in ∇(f − λg)(v1 ) = 0 otteniamo


2Av1 = 2λv1 , cioè Av1 = λv1 . Moltiplicando questa equazione scalarmente per v1 e
ricordando che kv1 k = 1, otteniamo

hv1 , Av1 i = λhv1 , v1 i = λ,

cioè f (v1 )(= λ1 ) = λ. Dunque Av1 = λ1 v1 , cioè il massimo λ1 di f |S n−1 è autovalore


di A con autovettore v1 .
b) Ottimizziamo ora la funzione f sull’insieme

S n−2 = {x ∈ S n−1 : hx, v1 i = 0},

cioè con i due vincoli g(x) = 0 e h(x) = hx, v1 i = 0. Sia λ2 = max f |S n−2 e sia
v2 ∈ S n−2 un punto di massimo, λ2 = f (v2 ) (esistenza garantita dal teorema di

26
Weierstrass). Per il teorema dei moltiplicatori di Lagrange esistono λ, µ ∈ R tali
che
∇(f − λg − µh)(v2 ) = 0.
Il gradiente di h è ∇h(x) = v1 , ∀x. Sostituendo quest’ultima insieme a ∇f (x) =
2Ax, ∇g(x) = 2x, si ottiene
2Av2 = 2λv2 + µv1 .
Moltiplicando questa equazione scalarmente per v1 e ricordando che hv1 , v2 i = 0 e
che A è simmetrica, otteniamo
2hv1 , Av2 i = 2hAv1 , v2 i = 2λ1 hv1 , v2 i = 0
= 2λhv1 , v2 i + µhv1 , v1 i = µ,
cioè µ = 0. Dunque Av2 = λv2 . Moltiplicando quest’ultima scalarmente per v2
otteniamo infine
hv2 , Av2 i = λhv2 , v2 i = λ,
cioè f (v2 )(= λ2 ) = λ. Dunque Av2 = λ2 v2 , cioè il massimo λ2 di f |S n−2 è autovalore
di A con autovettore v2 .
c) Ripetiamo il procedimento, ottimizzando la funzione f sull’insieme
S n−3 = {x ∈ S n−1 : hx, v1 i = 0, hx, v2 i = 0},
cioè con i tre vincoli g(x) = 0, h(x) = 0 e k(x) = hx, v2 i = 0. Sia λ3 = max f |S n−3
e sia v3 ∈ S n−3 un punto di massimo, λ3 = f (v3 ) (di nuovo l’esistenza del massimo
è garantita dal teorema di Weierstrass). Esistono allora λ, µ, ν ∈ R tali che
∇(f − λg − µh − νk)(v3 ) = 0,
cioè
2Av3 = 2λv3 + µv1 + νv2 .
Moltiplicando questa equazione scalarmente per v1 si trova µ = 0; moltiplicandola
per v2 si trova ν = 0. Dunque Av3 = λv3 . Moltiplicando questa scalarmente per
v3 si ottiene f (v3 )(= λ3 ) = λ, da cui Av3 = λ3 v3 . Ovviamente sarà λ1 ≥ λ2 ≥ λ3 e
v1 ⊥ v2 ⊥ v3 .
d) Iterando il procedimento otteniamo l’insieme
S n−k = {x ∈ S n−1 : hx, v1 i = hx, v2 i = · · · = hx, vk−1 i = 0}
tale che, posto λk = max f |S n−k = f (vk ), si ha Avk = λk vk . Gli ultimi 3 insiemi
saranno S 2 (sfera bidimensionale, per k = n − 2), S 1 (circonferenza, per k = n − 1)
e S 0 (per k = n), che consiste di 2 punti simmetrici rispetto all’origine, S 0 =
{vn , −vn }, essendo S 0 lo spazio ortogonale al vettore vn−1 in S 1 . Chiaramente f |S 0
è costante, essendo in generale f (−x) = f (x). Per dimostrare che l’ultimo vettore
vn (che è ortogonale a v1 , v2 , . . . , vn−1 ) è autovettore di A, basta osservare che
hvj , Avn i = hAvj , vn i = λj hvj , vn i = 0, ∀j = 1, 2, . . . , n − 1,

27
cioè il vettore Avn è ortogonale a tutti i vettori v1 , v2 , . . . , vn−1 , ed è quindi pro-
porzionale a vn , Avn = λvn . Da questa segue subito che λ = λn = f (vn ).
Per come sono stati definiti i vettori vk (usando i massimi ad ogni passo) si ha
λ1 ≥ λ2 ≥ · · · ≥ λn , e λ1 = f (v1 ) è il massimo autovalore di A, mentre λn = f (vn )
è il minimo autovalore di A. Questo metodo dimostra che tutti gli autovalori di A
sono reali essendo λk = f (vk ) ∈ R ∀k = 1, . . . , n. Il procedimento termina a S 0
perchè abbiamo costruito n vettori v1 , v2 , . . . , vn ortogonali tra loro e di norma 1,
cioè una base ortonormale di Rn costituita di autovettori di A.
Abbiamo dimostrato che il massimo di f su S n−1 coincide con il più grande au-
tovalore di A, max f |S n−1 = λ1 . È facile dimostrare che il minimo di f su S n−1
coincide con il più piccolo autovalore di A (cioè λn ). Infatti, posto λ̃1 = min f |S n−1
e indicando con ṽ1 ∈ S n−1 un punto di minimo, si trova (procedendo come al punto
a)) che Aṽ1 = λ̃1 ṽ1 , cioè il minimo di f su S n−1 è autovalore di A. Poichè tutti gli
autovalori di A sono dei valori assunti da f su S n−1 , necessariamente λ̃1 sarà il più
piccolo autovalore. In conclusione si ha che max f |S n−1 = massimo autovalore di A,
min f |S n−1 = minimo autovalore di A, e i punti di massimo e di minimo sono i cor-
rispondenti autovettori (normalizzati a 1).
32. Nell’esercizio 18 dei risolti si ha che, posto v = (x, y, z),
1 1
 
0 2 2
1 1
f (x, y, z) = xy + xz + yz = hv, Avi, dove A =  2
0 2
.
1 1
2 2
0
Per l’esercizio precedente, il massimo M e il minimo m di f sulla sfera unitaria
coincidono con il massimo e il minimo autovalore della matrice A. Il polinomio
caratteristico di A è

−λ 1 1
2 2
p(λ) = |A − λI| = 12 −λ 12 = − 14 4λ3 − 3λ − 1 .

1 1
−λ
2 2

Notando che p(1) = 0, possiamo scomporre con Ruffini ottenendo


2
p(λ) = −(λ − 1) λ + 21 ,

da cui M = 1, m = −1/2, in accordo con l’esercizio 18. Calcolando gli autovettori


(normalizzati a 1) troviamo che v = ( √13 , √13 , √13 ) è autovettore di λ = 1, mentre
qualsiasi vettore w = (x, y, z) ∈ S 2 tale che x+y +z = 0 è autovettore di λ = −1/2.
Questi coincidono con i punti di massimo e di minimo ottenuti nell’esercizio 18.
33. a) Indichiamo con x il vettore generico di Rn , x = (x1 , x2 , . . . , xn ). Essendo
x21 x22 x2n
g(x) = a21
+ a22
+ ··· + a2n
− 1,

si ha subito che
 
∇g(x1 , x2 , . . . , xn ) = 2 x1 x2
, , . . . , xa2n
a21 a22
= 2B −2 x,
n

28
dove B è la matrice diagonale
 
  1
a1 a21
1
a2
   
−2
 a22 
B=  =⇒ B =  .
 
... ..
.
   
 
an 1
a2n

Ricordando che ∇f (x) = 2Ax, il sistema ∇(f −λg)(x) = 0 diventa 2Ax = 2λB −2 x,
cioè moltiplicando a sinistra per B 2 , B 2 Ax = λx. Se x soddisfa il vincolo, molti-
plicando l’equazione Ax = λB −2 x scalarmente per x e notando che
x21 x22 x2n
hx, B −2 xi = a21
+ a22
+ ··· + a2n
= 1,

otteniamo hx, Axi = λ, cioè f (x) = λ.


b) Notiamo che si ha
B 2 A = B(BAB)B −1 ,
cioè la matrice B 2 A è simile alla matrice à = BAB. Ne segue che B 2 A e à hanno
lo stesso polinomio caratteristico in quanto

B 2 A − λI = B(BAB − λI)B −1 = B(Ã − λI)B −1

e dunque
2
B A − λI = Ã − λI .

Quindi B 2 A e à hanno gli stessi autovalori. Inoltre si ha Ãy = λy se e solo se

BABy = λy ⇔ (B 2 A)(By) = λ(By) ⇔ B 2 Ax = λx,

dove x = By.
c) Se x = By, cioè xj = aj yj , è immediato che
X x2 X
a2
j
= 1 ⇐⇒ yj2 = 1.
j

Inoltre poichè B è simmetrica, si ha

f (x) = hx, Axi = hBy, AByi = hy, BAByi = hy, Ãyi.

Da questo si deduce che ottimizzare f sull’ellissoide E n−1 è equivalente a ottimiz-


zare la funzione f˜(y) = hy, Ãyi sulla sfera unitaria S n−1 . Utilizzando i risultati
dell’esercizio 31, otteniamo che il massimo M di f |E n−1 è il più grande autovalore
di Ã, e il minimo m di f |E n−1 è il più piccolo autovalore di Ã.
34. Nell’esercizio 12 dei proposti si ha che, posto v = (x, y, z),
1 1
 
0 2 2
1 1
f (x, y, z) = xy + xz + yz = hv, Avi, dove A =  2
0 2
.
1 1
2 2
0

29
Dobbiamo ottimizzare questa forma quadratica sull’ellissoide

g(x, y, z) = x2 /4 + y 2 /4 + z 2 − 1 = 0.

Possiamo allora applicare i risultati dell’esercizio precedente con n = 3, a1 = a2 = 2,


a3 = 1. Detta B la matrice
 
2 0 0
B =  0 2 0 ,
0 0 1

è sufficiente calcolare il minimo e il massimo autovalore della matrice


0 12 12
     
2 0 0 2 0 0 0 2 1
à = BAB =  0 2 0   21 0 12   0 2 0  =  2 0 1  .
1 1
0 0 1 2 2
0 0 0 1 1 1 0

Il polinomio caratteristico di à è

−λ 2 1

p(λ) = Ã − λI = 2 −λ 1
= −λ3 + 6λ + 4.

1 1 −λ

Notando che p(−2) = 0 possiamo scomporre con Ruffini ottenendo



p(λ) = −(λ + 2)(λ2 − 2λ − 2) = 0 ⇐⇒ λ = −2, 1 ± 3.

Concludiamo che

M = max f |g=0 = 1 + 3, m = min f |g=0 = −2,

in accordo con i risultati dell’esercizio 12 (si veda anche l’esercizio 30).

35. Nell’esercizio 19 dei risolti, posto v = (x, y, z), si ha

f (x, y, z) = (x + y + z)2 = x2 + y 2 + z 2 + 2xy + 2xz + 2yz


 
1 1 1
= hv, Avi, dove A =  1 1 1  .
1 1 1

Dobbiamo ottimizzare questa forma quadratica sull’ellissoide g(x, y, z) = 1 dove


y2 z2
g(x, y, z) = x2 + 2y 2 + 3z 2 = x2 + „ «2 + „ «2 .
√1 √1
2 3

Applichiamo i risultati dell’esercizio 33 con n = 3, a1 = 1, a2 = √1 , a3 = √1 . Detta


2 3
B la matrice
1 0 0
 
1
B =  0 √2 0  ,
0 0 √13

30
dobbiamo calcolare il minimo e il massimo autovalore della matrice
1 0 0 1 0 0
   
1 1 1
1 1
à = BAB =  0 √2 0   1 1 1   0 √2 0 
0 0 √13 1 1 1 0 0 √13
 
1 √12 √13
=  √12 21 √16  . (6)
 
√1 √1 1
3 6 3

Il polinomio caratteristico di à è

√1 √1
1−λ

2 3
p(λ) = Ã − λI = √12 1
− λ √1 = −λ3 + 11 2
λ = −λ2 (λ − 11
).

2 6 6 6
√1 1 1
−λ


3
3 6

Gli autovalori di à sono quindi λ = 0, 11


6
, e concludiamo che
11
M = max f |g=1 = 6
, m = min f |g=1 = 0,

in accordo con i risultati dell’esercizio 19.

36. Applichiamo i risultati dell’esercizio 33 con n = 3, a1 = a, a2 = b, a3 = c. Il


massimo M e il minimo m di f (x, y, z) = xy + xz + yz vincolati all’ellissoide
2
x2 2
a2
+ yb2 + zc2 = 1 coincidono con il massimo e il minimo autovalore della matrice

0 12 12 0 ab ac
     
a 0 0 a 0 0 2 2
à = BAB =  0 b 0   21 0 12   0 b 0  =  ab 2
0 bc2  .
1 1 ac bc
0 0 c 2 2
0 0 0 c 2 2
0

Il polinomio caratteristico di à è

−λ ab 2
ac
2


p̃(λ) = Ã − λI = 2 −λ bc2
ab

ac bc
−λ
2 2
= − λ3 + 41 a2 b2 + a2 c2 + b2 c λ + 41 a2 b2 c2
2


= − 14 λ3 − a2 b2 + a2 c2 + b2 c2 λ − a2 b2 c2 = − 41 p(λ),
  

dove p(λ) è il polinomio dell’esercizio 28. Poichè Ã è una matrice simmetrica, i suoi
autovalori sono reali, dunque le radici di p(λ) sono tutte reali. Per il teorema dei
segni di Cartesio, poichè p(λ) ha una variazione e p(−λ) ha due variazioni, p(λ) ha
una radice positiva λ3 e due radici negative λ1 ≤ λ2 < 0. (Queste sono contate con
la loro molteplicità, quindi è incluso il caso di a = b = c in cui λ1 = λ2 , si veda
l’esercizio 29.) Dall’esercizio 33 si deduce che M = λ3 e m = λ1 , in accordo con i
risultati dell’esercizio 28.

31
37. Poichè l’ellissoide definito da
3
X
g(x) = hx, Axi = Aij xi xj = 1
i,j=1

è centrato nell’origine, dobbiamo determinare i punti sull’ellissoide a distanza mas-


sima e minima dall’origine. (La distanza massima è il semiasse maggiore, quella
minima è il semiasse minore.) Si tratta quindi di determinare gli estremi della
funzione distanza kxk, o del suo quadrato

f (x) = kxk2 = x21 + x22 + x23 ,

con il vincolo g(x) = 1. Si ha (vedi esercizio 31)

∇f (x) = 2x, ∇g(x) = 2Ax.

Sostituendo nel sistema ∇(f − λg)(x) = 0 otteniamo

2x = 2λAx =⇒ Ax = λ1 x.

Quindi se (x, λ) risolve il sistema


(
∇(f − λg)(x) = 0
g(x) = 1,

il punto critico vincolato x è autovettore di A e il moltiplicatore di Lagrange λ è il


reciproco 1/µ di un autovalore µ di A. Inoltre moltiplicando l’equazione x = λAx
scalarmente per x otteniamo

hx, xi = kxk2 = f (x) = λhx, Axi = λ,

cioè il moltiplicatore di Lagrange λ coincide con il valore di f nel punto critico


vincolato x. Siano µ1 , µ2 , µ3 gli autovalori di A, con 0 < µ1 ≤ µ2 ≤ µ3 (ricordiamo
che A è definita positiva). Da quanto detto sopra segue che
n o
M = max f |g=1 = max µ1 : µ è autovalore di A = µ11 ,
n o
1 1
m = min f |g=1 = min µ
: µ è autovalore di A = µ3
.

Poichè f (x) è la distanza al quadrato di x dall’origine, concludiamo che il semiasse


√ √
maggiore dell’ellissoide è 1/ µ1 , e il semiasse minore è 1/ µ3 .

32

Potrebbero piacerti anche